1answer.
Ask question
Login Signup
Ask question
All categories
  • English
  • Mathematics
  • Social Studies
  • Business
  • History
  • Health
  • Geography
  • Biology
  • Physics
  • Chemistry
  • Computers and Technology
  • Arts
  • World Languages
  • Spanish
  • French
  • German
  • Advanced Placement (AP)
  • SAT
  • Medicine
  • Law
  • Engineering
VladimirAG [237]
3 years ago
5

If Anna pays $18 for a pack of 25 markers how much is each marker

Mathematics
1 answer:
gayaneshka [121]3 years ago
6 0
18 / 25 = 0.72 per marker
You might be interested in
16k-100k polynomial
Alexeev081 [22]
=  -84k hope it helps
3 0
2 years ago
Larry uses 9 eggs to fill 3 baskets. If each basket has the same number of eggs, how many eggs will there be in 30 baskets?
Evgesh-ka [11]

Answer:

90 eggs

Step-by-step explanation:

9/3=x/30

3x=270

×90

7 0
3 years ago
Read 2 more answers
Hello, Brainly community!
ioda

Answer:

(B)  \displaystyle \frac{W(3.1) - W(2.9)}{0.2}

General Formulas and Concepts:

<u>Calculus</u>

Limits

Derivatives

  • The definition of a derivative is the slope of the tangent line.

Derivative Notation

Instantaneous Rates

  • Tangent Line: \displaystyle f'(x) = \frac{f(b) - f(a)}{b - a}

Step-by-step explanation:

Since we are trying to find a <em>rate</em> at which W(t) changes, we must find the <em>derivative</em> at <em>t</em> = 3.

We are given 2 close answer choices that would have the same <em>numerical</em> answer but different <em>meanings</em>:

  1. (A)  \displaystyle  \lim_{t \to 3} W(t)
  2. (B)  \displaystyle \frac{W(3.1) - W(2.9)}{0.2}

If we look at answer choice (A), we see that our units would simply just be volume. It would not have the units of a rate of change. Yes, it may be the closest numerically correct answer, but it does not tell us the <em>rate</em> at which the volume would be changing and it is not a derivative.

If we look at answer choice (B), we see that our units would be cm³/s, and that is most certainly a rate of change. Answer choice (B) is also a <em>derivative</em> at <em>t</em> = 3, and a derivative tells us what <em>rate</em> something is changing.

∴ Answer choice (B) will give us the best estimate for the value of the instantaneous rate of change of W(t) when <em>t</em> = 3.

Topic: AP Calculus AB/BC (Calculus I/I + II)

Unit: Differentiation

Book: College Calculus 10e

8 0
2 years ago
In AHIJ, h = 38 inches, i = 45 inches and j=61 inches. Find the measure of ZH to the nearest degree.
Paraphin [41]

Answer:

38°

Step-by-step explanation:

Delta math

7 0
2 years ago
Sean tried to drink a slushy as fast as he could. He drank the slushy at a constant rate. There were originally 275 milliliters
grandymaker [24]

Answer: A and B

Step-by-step explanatiOn correct one is the first page in the sentence scripture

3 0
3 years ago
Read 2 more answers
Other questions:
  • 30 POINTS!!!!
    7·1 answer
  • Use the elimination method to solve this system.
    8·1 answer
  • Compute r6r6, l6l6, and m3m3 to estimate the distance traveled over [0, 3] if the velocity at half-second intervals is as follow
    12·1 answer
  • What is the value of f(x) = 9x when x = -2? <br> i think its 1/81
    5·1 answer
  • Is 7 a factor of 46 please help me now or I am dead
    12·2 answers
  • Maria had 7 boxes. A week later there was a fire. After the fire she had 22 boxes how many boxes did Maria start with?
    14·1 answer
  • 1. To graph any linear equation in the calculator it needs to be in the slope
    10·1 answer
  • Y= -¹⁄₂₀ (x-25)² + 30
    13·1 answer
  • What is the value of x in the equation 25-0 25x=-3? 0 -22 0 -11 o 11 o 22​
    11·2 answers
  • Reflect the given triangle over the x axis. <br> [3 6 3<br> -3 3 3]
    15·1 answer
Add answer
Login
Not registered? Fast signup
Signup
Login Signup
Ask question!